Đến nội dung

Hình ảnh

BĐT AM-GM


  • Please log in to reply
Chủ đề này có 339 trả lời

#221
killerdark68

killerdark68

    Thượng sĩ

  • Thành viên
  • 266 Bài viết

Có: $a+b+c=6\geq 3\sqrt[3]{abc}\Rightarrow abc\leq 8$

$(1+\frac{1}{a^3})+(1+\frac{1}{b^3})+(1+\frac{1}{c^3})=(1+1+1)+(\frac{1}{a^3}+\frac{1}{b^3}+\frac{1}{c^3})\geq 3+\frac{3}{abc}\geq 3+\frac{3}{8}=\frac{27}{8}$

Dấu bằng khi $a=b=c=2$

Bạn xem lại đề

Đề đúng rồi đó bạn

A=$(1+\frac{1}{a^3})+(1+\frac{1}{b^3})+(1+\frac{1}{c^3})$

  =$1+(\frac{1}{a^3}+\frac{1}{b^3}+\frac{1}{c^3})+(\frac{1}{a^3b^3}+\frac{1}{c^3b^3}+\frac{1}{a^3c^3})+\frac{1}{a^3b^3c^3}$

Áp dụng bdt AM-GM có 

$\frac{1}{a^3}+\frac{1}{b^3}+\frac{1}{c^3}$$\geq \frac{3}{abc}$

$\frac{1}{a^3b^3}+\frac{1}{c^3b^3}+\frac{1}{a^3c^3}$$\geq \frac{3}{a^2b^2c^2}$

suy ra A $\geq$  1+$\frac{3}{abc}+\frac{3}{a^2b^2c^2}+{a^3b^3c^3}$=$(1+\frac{1}{abc})^3$

Lại có a+b+c$\geq 3\sqrt[3]{abc}\Rightarrow \frac{1}{abc}\geq (\frac{3}{a+b+c})^3=\frac{1}{8}$

$\Rightarrow (1+\frac{1}{abc})^3 \geq (1+\frac{1}{8})^3=\frac{729}{512}$ 

Dấu bằng khi $a=b=c=2$


Bài viết đã được chỉnh sửa nội dung bởi killerdark68: 18-07-2014 - 09:55


#222
phuongduy

phuongduy

    Lính mới

  • Thành viên
  • 2 Bài viết

Cho x,y,z là các số thực dương thay đổi và thỏa mãn điều kiện xyz=1. Tính giá trị nhỏ nhất của biểu thức
$P=\sum \frac{x^2(y+z)}{y\sqrt y+2z\sqrt z}$



#223
hoduchieu2001

hoduchieu2001

    Binh nhất

  • Thành viên
  • 21 Bài viết

1,CMR: vơi mọi sô thực dương a,b,c ta co:

(b+c)(c+a)(a+b)$\geq$8(b+c-a)(c+a-b)(a+b-c)

 

2, cho cac so abc không âm thoả mãn abc=1 cmr

       * (a+b)(b+c)(c+a) $\geqslant$2(a+b+c+1)

 

 

     *   (a+b)(b+c)(c+a)$\geqslant$ 4(a+b+c-1)


Bài viết đã được chỉnh sửa nội dung bởi hoduchieu2001: 15-10-2014 - 21:51


#224
rainbow99

rainbow99

    Sĩ quan

  • Thành viên
  • 386 Bài viết

câu 1:

áp dụng bất đẳng thức cosi cho 2 số thực dương ta có

2$\sqrt{(b+c-a)(c+a-b)}$$\leq$ b+c-a+c+a-b=2c

Tương tự 2$\sqrt{(c+a-b)(a+b-c)}\leq$2a

                2$\sqrt{(b+c-a)(a+b-c)}\leq$2b

Nhân theo vế ta được bất đẳng thức cần chứng minh



#225
hoduchieu2001

hoduchieu2001

    Binh nhất

  • Thành viên
  • 21 Bài viết

câu 1:

áp dụng bất đẳng thức cosi cho 2 số thực dương ta có

2$\sqrt{(b+c-a)(c+a-b)}$$\leq$ b+c-a+c+a-b=2c

Tương tự 2$\sqrt{(c+a-b)(a+b-c)}\leq$2a

                2$\sqrt{(b+c-a)(a+b-c)}\leq$2b

Nhân theo vế ta được bất đẳng thức cần chứng minh

hình như bạn nhầm rồi (a+b)(b+c)(c+a) 8(b+c-a)(c+a-b)(a+b-c) bạn đọc lại đề



#226
rainbow99

rainbow99

    Sĩ quan

  • Thành viên
  • 386 Bài viết

hình như bạn nhầm rồi (a+b)(b+c)(c+a) 8(b+c-a)(c+a-b)(a+b-c) bạn đọc lại đề

 mình nhầm. Cái này phải áp dụng thêm một lần bđt cosi nữa mới được

$(a+b)(b+c)(c+a)\geq 2\sqrt{ab}.2\sqrt{ac}.2\sqrt{bc}=8abc$

=> đpcm


Bài viết đã được chỉnh sửa nội dung bởi rainbow99: 16-10-2014 - 21:07


#227
Chung Anh

Chung Anh

    Sĩ quan

  • Thành viên
  • 420 Bài viết

 

2, cho cac so abc không âm thoả mãn abc=1 cmr

       * (a+b)(b+c)(c+a) $\geqslant$2(a+b+c+1)

 

 

   

Ta có $VT=(a+b+c)(ab+bc+ca)-abc=(a+b+c)(ab+bc+ca)-1$

 Áp dụng bdt AM-GM cho 3 số ko âm=>$ab+bc+ca \geq 3\sqrt[3]{(abc)^2}=3$

                                                             $a+b+c\geq 3\sqrt[3]{abc}=3$

=>$VT\geq3(a+b+c)-1=2(a+b+c)+(a+b+c)-1\geq2(a+b+c)+3-1=2(a+b+c)+2=2(a+b+c+1)$

=>$VT\geqVP$<=> a=b=c=1


Bài viết đã được chỉnh sửa nội dung bởi Chung Anh: 23-11-2014 - 18:23

Chung Anh


#228
Van Chung

Van Chung

    Thượng sĩ

  • Thành viên
  • 212 Bài viết

2, cho cac so abc không âm thoả mãn abc=1 cmr

     *   (a+b)(b+c)(c+a)$\geqslant$ 4(a+b+c-1)

Ta có:

$ VT=(a+b)(b+c)(c+a)=(a+b+c)(ab+bc+ca)-abc$(*)

Áp dụng bđt Cauchy cho 3 số không âm ta có:

$a+b+c \geq 3\sqrt[3]{abc}=3(1)$

$ab+bc+ca \geq 3\sqrt[3]{(abc)^2}=3(2)$

Áp dụng (2) vào (*) ta được 

$VT \geq 3(a+b+c)-abc=4(a+b+c)-(a+b+c)-1 \geq 4(a+b+c) -3-1 \geq 4(a+b+c-1)$  ( theo (2))

$\Rightarrow đpcm$


Bài viết đã được chỉnh sửa nội dung bởi Van Chung: 08-12-2014 - 20:53

                    What doesn't kill you makes you stronger


#229
HoangHuyltt

HoangHuyltt

    Lính mới

  • Thành viên
  • 5 Bài viết

Mọi người cho mình hỏi bài này với

cho 3 số a,b,c dương thoả mãn 2a+4b+7c <= 2abc.tìm min P=a+b+c.



#230
babystudymaths

babystudymaths

    Sĩ quan

  • Thành viên
  • 312 Bài viết

Ta có:

$ VT=(a+b)(b+c)(c+a)=(a+b+c)(ab+bc+ca)-abc$(*)

Áp dụng bđt Cauchy cho 3 số không âm ta có:

$a+b+c \geq 3\sqrt[3]{abc}=3(1)$

$ab+bc+ca \geq 3\sqrt[3]{(abc)^2}=3(2)$

Áp dụng (2) vào (*) ta được 

$VT \geq 3(a+b+c)-abc=4(a+b+c)-(a+b+c)-1 \geq 4(a+b+c) -3-1 \geq 4(a+b+c-1)$  ( theo (2))

$\Rightarrow đpcm$

4(a+b+c)-(a+b+c)-1 \geq 4(a+b+c) -3-1 .vô lí vì

$a+b+c\geq 3=> -(a+b+c)\leq -3$


TLongHV


#231
babystudymaths

babystudymaths

    Sĩ quan

  • Thành viên
  • 312 Bài viết

Mọi người cho mình hỏi bài này với

cho 3 số a,b,c dương thoả mãn 2a+4b+7c <= 2abc.tìm min P=a+b+c.

 từ giả thiết =>$c\geq \frac{2a+4b}{2ab-7}$

=>$P\geq a+b+\frac{2a+4b}{2ab-7} <=>P\geq a+\frac{2ab-7}{2a} +(\frac{2a+4b}{2ab-7}-\frac{2}{a})+\frac{11}{2a} <=>P\geq a+\frac{11}{2a}+\frac{2ab-7}{2a}+\frac{2a^{2}+14}{a(2ab-7)}=> P\geq a+\frac{11}{2a}+\frac{2\sqrt{a^{2}+7}}{a}$

áp dụng bdt bunhia cho $(a,\sqrt{7}), (3,\sqrt{7})$, ta có

$P\geq a+\frac{11}{2a}+\frac{3a+7}{2a}=>P\geq a+\frac{9}{a}+\frac{3}{2}\geq \frac{15}{2} MinP=\frac{15}{2}<=>a=3, b=\frac{5}{2}, c=2$

(làm hơi tắt)


TLongHV


#232
Coppy dera

Coppy dera

    Sĩ quan

  • Thành viên
  • 334 Bài viết

a^2


Like đi  :like  :like  :like  :like  :like  :like  :like  :like  :like  :like  :like  :like  :like  :like 

 

Kết bạn qua facebook https://www.facebook.com/tqt2001


#233
Coppy dera

Coppy dera

    Sĩ quan

  • Thành viên
  • 334 Bài viết

P\geq a+b+\frac{2a+4b}{2ab-7} <=>P\geq a+\frac{2ab-7}{2a} +(\frac{2a+4b}{2ab-7}-\frac{2}{a})+\frac{11}{2a} <=>P\geq a+\frac{11}{2a}+\frac{2ab-7}{2a}+\frac{2a^{2}+14}{a(2ab-7)}=> P\geq a+\frac{11}{2a}+\frac{2\sqrt{a^{2}+7}}{a}............ :nav:  

 

Chú ýCách gõ công thức Toán.


Bài viết đã được chỉnh sửa nội dung bởi hachinh2013: 17-04-2015 - 09:22

Like đi  :like  :like  :like  :like  :like  :like  :like  :like  :like  :like  :like  :like  :like  :like 

 

Kết bạn qua facebook https://www.facebook.com/tqt2001


#234
Ngoc Hung

Ngoc Hung

    Đại úy

  • Điều hành viên THCS
  • 1547 Bài viết

Cho x, y, z > 0 và x + y +z = 6. Chứng minh rằng $\left ( 1+\frac{1}{x^{3}} \right )\left ( 1+\frac{1}{y^{3}} \right )\left ( 1+\frac{1}{z^{3}} \right )\geq \frac{729}{512}$



#235
nguyenhongsonk612

nguyenhongsonk612

    Thượng úy

  • Thành viên
  • 1451 Bài viết

Cho x, y, z > 0 và x + y +z = 6. Chứng minh rằng $\left ( 1+\frac{1}{x^{3}} \right )\left ( 1+\frac{1}{y^{3}} \right )\left ( 1+\frac{1}{z^{3}} \right )\geq \frac{729}{512}$

Áp dụng BĐT Holder ta có

$VT=\begin{pmatrix} \frac{1}{2}+\frac{1}{2}+\frac{1}{x^3} \end{pmatrix}\begin{pmatrix} \frac{1}{2}+\frac{1}{2}+\frac{1}{y^3} \end{pmatrix}\begin{pmatrix} \frac{1}{2}+\frac{1}{2}+\frac{1}{z^3} \end{pmatrix}$

$\geq \begin{pmatrix} \frac{1}{2}+\frac{1}{2}+\frac{1}{xyz} \end{pmatrix}^3\geq \begin{pmatrix} 1+\frac{1}{\begin{pmatrix} \frac{x+y+z}{3} \end{pmatrix}^3} \end{pmatrix}^3=\frac{729}{512}$

Dấu "=" xảy ra khi $x=y=z=2$


"...Từ ngay ngày hôm nay tôi sẽ chăm chỉ học hành như Stardi, với đôi tay nắm chặt và hàm răng nghiến lại đầy quyết tâm. Tôi sẽ nỗ lực với toàn bộ trái tim và sức mạnh để hạ gục cơn buồn ngủ vào mỗi tối và thức dậy sớm vào mỗi sáng. Tôi sẽ vắt óc ra mà học và không nhân nhượng với sự lười biếng. Tôi có thể học đến phát bệnh miễn là thoát khỏi cuộc sống nhàm chán khiến mọi người và cả chính tôi mệt mỏi như thế này. Dũng cảm lên! Hãy bắt tay vào công việc với tất cả trái tim và khối óc. Làm việc để lấy lại niềm vui, lấy lại nụ cười trên môi thầy giáo và cái hôn chúc phúc của bố tôi. " (Trích "Những tấm lòng cao cả")

~O) 


#236
trubatgioi

trubatgioi

    Binh nhất

  • Thành viên
  • 20 Bài viết

2,$\frac{a+b+c+d}{\sqrt[4]{abcd}}+\frac{16}{(a+b)(b+c)(c+d)(d+a)}\geq 5$
-Chứng minh :chú ý rằng $\frac{a+b+c+d}{\sqrt[4]{abcd}}=\frac{a+b}{2\sqrt[4]{abcd}}+\frac{b+c}{\sqrt[4]{abcd}}+\frac{c+d}{\sqrt[4]{abcd}}+\frac{d+a}{\sqrt[4]{abcd}}$
đến đây, nếu trở ại bài toán,ta chỉ cần áp dụng BĐT AM-GM cho 5 số là xong
(cái mình muốn nói ở đây là viêc tách hạng tử
nhiều bạn chắc chắn sẽ thắc mắc tại sao lại tách như trên .Trong tầm kiến thức ở AM-GM(tức là ta chỉ xem xét các khả năng giải bằng AM-GM cho bài toán trên ),mình xin giải thích như sau
+,nếu áp dụng trực tiếp AM-GM ,chắc chắn sẽ không đem lại kết quả rồi
+,chú ý rằng bài toán trên không hề cho đk ẩn(ngoài không âm)nên ta khả dĩ không thể áp dụng AM-GM ĐỂ RỒI ĐƯA VẾ TRÁI VỀ 1 BIỂU THỨC CÒN ẨN
+,từ đánh giá thứ 2,ta buộc phải tìm cách triệt tiêu hoàn toàn tất cả ẩn sau khi đã áp dụng AM-GM
tức là phải tách 1 trong 2 hạnh tử trên ra thành tổng của 1 số hạng tử náo đó
+,Phân thức $\frac{16abcd}{(a+b)(b+c)(c+d)(d+a)}$ tuyệt vời này chắc cũng chẳng ai tìm cách tách nó,vì vậy ta tìm cách tách phân thức $\frac{a+b+c+d}{\sqrt[4]{abcd}}$
+'dựa vào phân thức cố định trên,ta phải tách phân thức $\frac{a+b+c+d}{\sqrt[4]{abcd}}$ thành tổng các phân thức có mẫu là $k(a+b),k(b+c)..$ để triệt tiêu mẫu(số pphân thức tách được là 4 )
+,chọn k= 1(vì sao?chẳng vì sao cả,nếu không dc ta lại chọn số khác ,cho dễ tihs thôi) ta dc 1 dạng $\frac{a+b}{x}+\frac{b+c}{y}+\frac{c+d}{z}+\frac{d+a}{t}$
+,chọn x=y=z=t ,ta dc tổng trên bằng $\frac{2(a+b+c+d)}{x}$ vậy hiển nhiên $x=2\sqrt[4]{abcd}$
đến đây xem như ta đã hoàn thành trọn vẹn bài toán
 

hình như thiếu điều kiện abcd<=1 thì pải



#237
anh690136

anh690136

    Lính mới

  • Thành viên
  • 7 Bài viết

Mình xin đóng góp 1 bài:

 

$\bg_green \sqrt{\frac{a^{2}+2b^{2}}{a^{2}+ab+bc}} + \sqrt{\frac{b^{2}+2c^{2}}{b^{2}+bc+ca}} + \sqrt{\frac{c^{2}+2a^{2}}{c^{2}+ca+ab}} \geq 3$



#238
toanhoc12345

toanhoc12345

    Binh nhì

  • Thành viên
  • 19 Bài viết

cho con hỏi bài này ạ:

 

 

 

                    cho a+b+c =3.GTNN:

 

                                 S=  $\frac{a}{b.b+1}$ + $\frac{b}{c.c+1}$ + $\frac{c}{a.a+1}$

   

                con cám ơn

 



#239
Quoc Tuan Qbdh

Quoc Tuan Qbdh

    DragonBoy

  • Điều hành viên THCS
  • 1005 Bài viết

4,$\frac{a^2}{b+c}+\frac{b^2}{c+a}+\frac{c^2}{a+b}\geq \frac{a+b+c}{2}$
 

Dùng Svacxo $\sum \frac{a^{2}}{b+c}\geq \frac{(\sum a)^{2} }{2\sum a}=\frac{\sum a}{2}$

Dùng Cauchy: $\sum (\frac{a^{2}}{b+c}+(b+c))\geq \sum 2a$



#240
Quoc Tuan Qbdh

Quoc Tuan Qbdh

    DragonBoy

  • Điều hành viên THCS
  • 1005 Bài viết

Cho x,y,z la cac so thuc duong thoa man: $x^{3}+y^{3}+z^{3}=1$

Tim min : $\frac{x^{2}}{\sqrt{1-x^{2}}}+\frac{y^{2}}{\sqrt{1-y^{2}}}+\frac{z^{2}}{\sqrt{1-z^{2}}}$

$\sum \frac{x^{2}}{\sqrt{1-x^{2}}}=\sum \frac{x^{3}}{x\sqrt{1-x^{2}}}\geq 2\sum x^{3}=2$

 

 






1 người đang xem chủ đề

0 thành viên, 1 khách, 0 thành viên ẩn danh